Sunteți pe pagina 1din 10

and

P(A brown-eyed, and a blue-eyed parent have a blue-eyed child) =


1
2
_

+
_
.
Problem 25. (Communication through a noisy channel) A transmitter wishes
to send one of two alternative messages, a or b. Because of the way these messages
originate, it is known that b messages are twice as likely as a messages. These mes-
sages are encoded into binary messages (codewords), in order to be transmitted over
a digital channel. Assume that the two codewords are 0110 and 1000, respectively.
However, the channel is noisy and each bit transmitted may be received incorrectly,
according to the probabilities shown below. For example, if a zero is sent, there is prob-
ability 0.2 that a 1 is received. Assume that errors during the transmission of dierent
bits are statistically independent. Given that the receiver received the sequence 0001,
nd the probability that message a was transmitted.
Problem 26. We are given three coins. The rst coin is a fair coin painted blue
on the head side and white on the tail side. The other two coins are biased so that the
probability of a head is p. They are painted blue on the tail side and red on the head
side. Two of the three coins are to be selected at random and tossed. Describe the
outcomes in the sample space. It was experimentally determined that the probability
that the sides that land face up are of the same color is 29/96. What are the possible
values of p?
Solution. There are two stages to the experiment: the selection of the two coins and
the ipping of the coins. There are three dierent ways that two coins can be selected:
the 1st and 2nd, the 1st and 3rd, and the 2nd and 3rd. Each of these pairs are equally
likely to be selected. For each pair, the ips have four possible outcomes: (heads,
heads), (heads, tails), (tails, heads), (tails, tails). The sample space can be described
as follows:
The probability that the sides that land face up are the same is then:
P(same) = P
_
(blue,blue)
_
+P
_
(red,red)
_
=
1
3
_
1
2
(1 p) +
1
2
(1 p) +p
2
+ (1 p)
2
_
=
1
3
(2p
2
3p + 2).
Setting this equal to
29
96
and solving the quadratic equation for p yields p = 5/8 or
p = 7/8.
Problem 27. A certain test for a disease is only 60% accurate. In order to arrive
at a somewhat trustworthy result, a blood sample is repeatedly tested until there are
4 test outcomes that indicate the same result. Given that a particular blood sample
came from a patient carrying the disease, nd the probability that in exactly i tests,
for i = 4, 5, 6, 7, the patients blood will be found to have the disease.
14
Homework 3.2
1/3
1/3
1 & 2
1/3
1 & 3
heads, tails
tails, heads
(1/2)p
(1/2)(1-p)
white, red
(1/2)(1-p)
(1/2)p
tails, tails
heads, heads
2 & 3
heads, tails
tails, heads
heads, heads
tails, tails
p(1-p)
(1-p)p
(1-p)(1-p)
pp
blue, red
blue, blue
red, blue
red, red
heads, tails
tails, heads
(1/2)p
(1/2)(1-p)
white, red
(1/2)(1-p)
(1/2)p
heads, heads
tails, tails white, blue
blue, blue
blue, red
white, blue
blue, blue
blue, red
*
*
*
*
Figure 0.7:
15
Solution.We use the binomial probabilities:
P(correct diagnosis in 4 tests) =
_
4
4
_
(0.6)
4
= 0.1296,
P(correct diagnosis in 5 tests) =
_
5
4
_
(0.6)
4
(0.4)
1
= 0.2592,
P(correct diagnosis in 6 tests) =
_
6
4
_
(0.6)
4
(0.4)
2
= 0.31104,
P(correct diagnosis in 7 tests) =
_
7
4
_
(0.6)
4
(0.4)
3
= 0.29.
Problem 28. A new robot is equipped with the brains to solve any kind of problem
set! Due to bugs in the design, however, and independent of the diculty of problems,
this robot gets problems right with probability 0.65, independently for each problem.
This robot is very expensive, and you have to decide whether to keep it or not. You
decide that if at any point the number of wrong problems exceeds the number of
correct problems by more than 15, you will return the robot. If, however, at any point
the number of correct problems exceeds the number of incorrect problems by 15, then
you will keep the robot. What are the chances that you will return the robot? Hint:
Use the solution to the gamblers ruin problem.
Solution. We can reduce this problem to the gamblers ruin problem with N = 30, k =
15, and p = 0.65. The event equivalent to returning the robot is that of G1 winning.
The probability of this event is
P15 =
1 (0.35/0.65)
15
1 (0.35/0.65)
30
9.3 10
5
.
Problem 29. A particular jury consists of 7 jurors. Each juror has a 0.2 chance of
making the wrong decision, independently of the others. If the jury reaches a decision
by majority rule, what is the probability that it will reach a wrong decision?
Solution.The number of jurors that make the wrong decision can be modeled as a
binomial random variable with parameters n = 7 and p = .2. The jury as a whole
will make the wrong decision if 4, 5, 6, or 7 jurors make the wrong decision. Denote
these events by A, B, C, D, respectively. Since these events are mutually exclusive,
the probability of their union is the sum of their probabilities, so
P(Jury Error) = P(A) +P(B) +P(C) +P(D)
=
_
7
4
_
(0.2)
4
(0.8)
3
+
_
7
5
_
(0.2)
5
(0.8)
2
+
_
7
6
_
(0.2)
6
(0.8)
1
+
_
7
7
_
(0.2)
7
(0.8)
0
= 0.033.
16
Homework 3.3
Problem 30. Galtons quincunx: Consider a mechanical device in which balls are
dropped through a triangular array of nails, starting at the top. The nails are placed
in rows, with each nail having two nails symmetrically placed in the row underneath
it. There is a total of n rows. Every time a ball hits a nail it has a probability 0.5 to
fall to the left of the nail and a probability of 0.5 to fall to the right of the nail. The
ball comes to rest at the nth row. What is the probability that the ball rests to the
left of the kth nail of the nth row?
Problem 31. Calculating the odds. Let A be an event such that 0 < P(A) < 1.
The odds in favor of A are dened to be
O(A) =
P(A)
P(A
c
)
,
while the odds against A are dened to be the reciprocal of O(A). [To connect the term
odds with its common usage, note for example that if the probability that a given
horse wins a race at the track is 1/3, the odds against the horse winning are 2 to 1. A
fair racetrack would then pay $2 for every $1 bet on the horse (plus the original $1
bet), if the horse wins; fair here means that the racetrack would break even on the
average this will become more precise in Chapter 2, when we will discuss the notion of
expected value.] This problem deals with a formula for calculating conditional odds,
that is, odds based on some partial information. If A and B are events with P(A) > 0
and P(B) > 0, the odds in favor of A given B are dened as
O(A| B) =
P(A| B)
P(A
c
| B)
.
Show that
O(A| B) = L(B| A)O(A),
where L(B| A) is the so called likelihood ratio of B given A, dened as
L(B| A) =
P(B| A)
P(B| A
c
)
.
Solution.By denition, O(A| B) is equal to
P(A| B)
P(A
c
| B)
=
P(A B)/P(B)
P(A
c
B)/P(B)
=
P(A B)
P(A
c
B)
=
P(B| A)P(A)
P(B| A
c
)P(A
c
)
= L(B| A)O(A).
Problem 32. Hypothesis testing. May B. Lucky is a compulsive gambler who
is convinced that on any given day she is either lucky, in which case she wins each
red/black bet she makes in the roulette with probability pL > 1/2, or she is unlucky,
in which case she wins each red/black bet she makes in the roulette with probability
pU < 1/2. May visits the casino every day, and believes that she knows the a priori
probability that any one given visit is a lucky one (i.e., corresponds to pL rather
than pU). To improve her chances, May adopts a system whereby she estimates on-line
whether she is lucky or unlucky on a given day, by keeping a running count of the
numbers of bets that she wins and loses. In particular, she continues to play until the
17
conditional odds in favor of the event {lucky on the current day}, given the number
of wins and losses so far, fall below a certain threshold (see the preceding problem).
As soon as this happens, she stops playing. Provide a simple algorithm for updating
Mays conditional odds with each play. Note: This example is typical of reasoning in
sequential hypothesis testing systems, where the probability of correctness of a certain
hypothesis, given some evidence, is calculated and sequentially updated.
Solution.Let A be the event that May is lucky on the current day, and let Bm,n be the
event that m wins and n losses have occurred so far. We assume independence of the
results of dierent spins/plays. Then we have, using the odds formula of the preceding
problem and the binomial formula,
O(A| Bm,n)
O(A)
= L(Bm,n | A) =
P(Bm,n | A)
P(Bm,n | A
c
)
=
_
m+n
m
_
p
m
L
(1 pL)
n
_
m+n
m
_
p
m
U
(1 pU)
n
=
_
pL
pU
_
m
_
1 pL
1 pU
_
n
From this formula, a convenient recursive algorithm is obtained. After m+n plays, in
which m wins and n losses occurred, we have
O(A| Bm+1,n) = O(A| Bm,n)
pL
pU
, if she wins in the next play,
O(A| Bm,n+1) = O(A| Bm,n)
1 pL
1 pU
, if she loses in the next play.
The initial condition is O(A| B0,0) is equal to the initial (unconditional) odds O(A)
(which May knows by assumption).
Problem 33. * Let A and B be events such that A B. Can A and B be
independent?
Solution.The events A and B are independent if and only if P(A)P(B) = P(A B) =
P(A), where the last equality follows from the fact that A B. This can be the case
if and only if P(A) = 0 or P(B) = 1.
Problem 34. * Alice starts by ipping a coin until she obtains tails for the rst time.
After that, Bob starts ipping until he obtains tails for the rst time, and they keep
alternating similarly. Assume that coin ips are independent and that the probability
of heads at each ip is p. The game ends when either Alice has accumulated m heads,
in which case she wins, or when Bob has accumulated n heads, in which case he wins,
whichever comes rst. Show that the probability qm,n that Alice wins satises
qm,n = pqm1,n + (1 p)(1 qn,m).
Solution.
SECTION 1.6. Counting
Problem 35. A parking lot contains 100 cars that all look quite nice from the
outside. However, K of these cars happen to be lemons. The number K is known to
lie in the range {0, 1, . . . , 9}, with all values equally likely.
18
Homework 3.4
(a) We testdrive 20 distinct cars chosen at random, and to our pleasant surprise, none
of them turns out to be a lemon. Given this knowledge, what is the probability
that K = 0?
(b) Repeat part (a) when the 20 cars are chosen with replacement; that is, at each
testdrive, each car is equally likely to be selected, including those that were
selected earlier.
Solution.(a) Let A be the event that all 20 cars tested are good. We are asked to nd
P(K = 0 | A). Using Bayes rule, we have
P(K = 0 | A) =
P(K = 0)P(A| K = 0)

9
i=0
P(K = i)P(A| K = i)
.
It is given that P(K = i) = 1/10 for all i. To compute P(A| K = i), we condition
on the event of exactly i lemons, and reason as follows. The rst selected car has
probability (100 i)/100 of being good. Having succeeded in the rst selection, we are
left with 99 cars out of which i are lemons; thus, the second selected car has probability
(99 i)/99 of being good. Continuing similarly, and using the multiplication rule, we
obtain
P(A| K = i) =
(100 i)(99 i) (81 i)
100 99 81
,
from which we can then obtain P(K = 0 | A).
(b) We use the exact same argument as in part (a), except that we need to recalculate
P(A| K = i). Since the cars are chosen with replacement, we are dealing with 20
independent Bernoulli trials. The probability of nding a good car in any one trial is
(100 i)/100. The probability of nding good cars in all 20 trials is
P(A| k = i) =
_
100 i
100
_
20
,
from which we can then obtain P(K = 0 | A).
Problem 36. A certain department oers 8 lower level courses: {L1, L2, . . . , L8}
and 10 higher level courses: {H1, H2, . . . , H10}. A valid curriculum consists of 4 lower
level courses, and 3 higher level courses.
(a) How many dierent curricula are possible?
(b) Suppose that {H1, . . . , H5} have L1 as a prerequisite and {H6, . . . H10} have L2
and L3 as prerequisites, i.e., any curricula which involve, say, one of {H1, . . . , H5}
must also include L1. How many dierent curricula are there?
Solution.(a) There are
_
8
4
_
ways to pick 4 lower level classes, and
_
10
3
_
ways to choose
3 higher level classes, so there are
_
8
4
__
10
3
_
valid curricula.
(b) This part is more involved. We need to consider several dierent cases
19
(i) Suppose we do not choose L1. Then both L2 and L3 must be chosen; otherwise
no higher level courses would be allowed. Thus, we need to choose 2 more lower
level classes out of the remaining 5, and 3 higher level classes from the available
5. We then obtain
_
5
2
__
5
3
_
valid curricula.
(ii) If we choose L1 but choose neither L2 nor L3, we have
_
5
3
__
5
3
_
choices.
(iii) If we choose L1 and choose one of L2 or L3, we have 2
_
5
2
__
5
3
_
choices. This is
because there are two ways of choosing between L2 and L3,
_
5
2
_
ways of choosing
2 lower level classes from L3, . . . , L8, and
_
5
3
_
ways of choosing 3 higher level
classes from H1, . . . , H5.
(iv) Finally, if we choose L1, L2, and L3, we have
_
5
1
__
10
3
_
choices.
Note that we are not double counting, because there is no overlap in the cases we are
considering, and furthermore we have considered every possible choice. The total is
obtained by adding the counts for the above four cases.
Problem 37. How many 6-word sentences can be made using each of the 26 letters
of the alphabet exactly once? A word is dened as a nonempty (possibly jibberish)
sequence of letters.
Solution.Think of a 6-word sequence using the 26 letters as a sequence of length 31:
there are 26 letters, plus ve blanks to separate one word from the next. Note that
the blanks cannot be at the beginning or the end of the sequence, which only leaves 29
available positions. Thus, there are
_
29
5
_
possible choices for the locations of the blanks.
Then, the 26 letters can be placed in arbitrary order (permutation) in the remaining
26 locations (26! choices). Thus, the number of possible sentences is 26!
_
29
5
_
.
Problem 38. A candy factory has an endless supply of red, orange, yellow, green,
blue, and violet jelly beans. The factory packages the jelly beans into jars of 100 jelly
beans each. One possible color distribution, for example, is a jar of 56 red, 22 yellow,
and 20 green jelly beans. As a marketing gimmick, the factory guarantees that no two
jars have the same color distribution. What is the maximum number of jars the factory
can produce?
Solution.Think of lining up the jelly beans, by rst placing the red ones, then the orange
ones, etc. We also place 5 dividers to indicate where one color ends and another starts.
(Note that two dividers can be adjacent if there are no jelly beans of some color.) By
considering both jelly beans and dividers, we see that there is a total of 105 positions.
Choosing the number of jelly beans of each color is the same as choosing the positions
of the dividers. Thus, there are
_
105
5
_
possibilities, and this is the number of possible
jars.
Problem 39. We have m married couples (2m individuals). After a number of
years, each person has died, independently, with probability p. Let N be the number
of surviving individuals. Let C be the number of couples in which both individuals are
alive. Find the conditional probability P(C = c | N = n).
Solution.Use Bayes rule. Find P(N = n| C = c) rst. The c couples give 2c alive
individuals. Then nd the number of ways that we can have n 2c alive ones in the
remaining couples without having any alive couples.NEEDS TO BE WORKED OUT
- TRICKY
20
Problem 40. Consider three independent rolls of a fair six-sided die.
(a) What is the probability that the sum of the three rolls is 11?
(b) What is the probability that the sum of the three rolls is 12?
(c) In the seventeenth century, Galileo explained the experimental observation that
a sum of 10 is more frequent than a sum of 9, even though both 10 and 9 can be
obtained in six distinct ways. Can you retrace Galileos thinking?
Solution. (a) The number of outcomes that leads to a sum of 11 is the number of
outcomes of the rst two rolls that lead to a sum greater than or equal to 11 6 = 5
and less than or equal to 11 1 = 10. The number of outcomes of the rst two rolls
that leads to sum less than 5 or greater than 10 is 9. So the desired probability is
6
2
9
6
3
.
(b) The number of outcomes that leads to a sum of 12 is the number of outcomes of
the rst two rolls that lead to a sum greater or equal to 12 6 = 6 and less than or
equal to 12 1 = 11. The number of outcomes of the rst two rolls that leads to sum
less than 6 or greater than 11 is 11. So the desired probability is
6
2
11
6
3
.
(c) Each of the sums 9 and 10 can be obtained in six distinct ways:
9 = 1 + 2 + 6 = 1 + 3 + 5 = 1 + 4 + 4 = 2 + 2 + 5 = 2 + 3 + 4 = 3 + 3 + 3,
10 = 1 + 3 + 6 = 1 + 4 + 5 = 2 + 2 + 6 = 2 + 3 + 5 = 2 + 4 + 4 = 3 + 3 + 4.
However, the number of outcomes that sum to 9 is 25, while the number of outcomes
that sum to 10 is 27. Thus, a sum of 10 has probability 27/6
3
and is more frequent
than a sum of 9, which has probability 25/6
3
.
Problem 41. The weather on any given day can be sunny, cloudy, rainy, or snowy.
Assume that a snowy day can happen only during the winter, that a rainy day cannot
happen in the summer, and that each season has 90 days. What is the number of all
possible distinct 360-day weather sequences?
Solution. The number of possible sequences is (4 1)
90
4
90
(4 1)
90
(4 2)
90
.
Problem 42. Consider a backgammon match with 25 games, each of which can have
one of two outcomes: win (1 point), or loss (0 points). Find the number of all possible
distinct score sequences under the following alternative assumptions.
(a) All 25 games are played.
(b) The match is stopped when one player reaches 13 points.
Solution. (a) 2
25
.
(b) First note that under this rule, each match will be stopped after a number of games
ranging from 13 to 25. If a match will be stopped at the kth game with player 1 having
21
Homework 3.5
13 points, then the last game was a win and k 13 of the previous games was a loss.
So, there are
_
k1
k13
_
matches that ends at the kth game with player 1 having a score
of 13. Taking player 2 into consideration and summing over k, we obtain
2
25

k=13
_
k 1
k 13
_
possible distinct score sequences.
Problem 43. Alice and Bob each have a deck of playing cards. Each turns over a
randomly selected card. Determine the following:
(a) The probability that the two cards are of the same suit.
(b) The probability that at least one card is a spade.
(c) The probability that neither card is a diamond.
(d) The probability that both cards are hearts or spades.
Solution. (a)
P(the two cards are of the same suit) = 4
_
13
52
_
2
= 4
_
1
4
_
2
.
(b)
P(at least one card is a spade) = 1 P(neither card is a spade)
= 1
_
3 13
52
_
2
= 1
_
3
4
_
2
(c)
P(neither card is a diamond) =
_
3 13
52
_
2
=
_
3
4
_
2
.
(d)
P(both cards are hearts or spades) =
_
2 14
52
_
2
=
_
1
2
_
2
.
Problem 44. A parking lot contains 100 cars, k of which happen to be lemons. N
of these cars are randomly selected and taken for a testdrive. Find the probability that
n of these cars turn out to be lemons.
Solution.Clearly if n > N or if n > k the probability must be zero. If n N and n k,
then we can nd the probability that the testdrive found n of the 100 cars defective by
counting the total number of size N subsets, and then the number of size N subsets
that contain n lemons. Clearly, there are
_
100
N
_
dierent subsets of size N. To count the
number of size N subsets with n lemons, we rst choose the lemons from the k total
lemons, and then choose the N n good cars from the remaining 100 k cars. Thus
the number of ways to choose a subset of size N from 100 cars, and get n lemons, is
_
k
n
_

_
100 k
N n
_
22
Homework 3.6
Homework 3.7
and thus the desired probability is
_
k
n
_

_
100k
Nn
_
_
100
N
_
Problem 45. Ninety students, including Joe and Jane, are to be split into three
classes of equal size, and this is to be done at random. What is the probability that
Joe and Jane end up in the same class?
Solution. Suppose we label the classes A, B, and C. The probability that Joe and
Jane will both be in class A is the number of possible combinations for class A that
involve both Joe and Jane, divided by the total number of combinations for class A.
Therefore the desired probability is
_
88
28
_
_
90
30
_.
Since there are three classrooms, the probability that Joe and Jane end up in the same
classroom is simply three times the answer we found above:
3
_
88
28
_
_
90
30
_.
Problem 46. Find the probability of winning in the following lottery. You choose
5 distinct integers in the range from 1 to 100. Then, the lottery operator chooses
randomly 10 distinct integers in the same range (all outcomes being equally likely).
You win if all of your 5 numbers are among those chosen by the operator.
Problem 47. A bank has a vault with a combination lock. The lock has a combina-
tion that consists of 10 integers in increasing order ranging from 1 to 90, but will open
if any 8 of the numbers are entered. Find the probability that a burglar will open the
vault on the rst try.
Solution.To nd the probability, we will nd the number of favorable outcomes, and
divide by the total number of possible outcomes. There are
_
10
8
_
favorable outcomes,
i.e., successful combinations that will open the lock. There are
_
90
8
_
total number of
ways to choose 8 numbers out of 90, and therefore the probability that the burglar will
open the vault on his rst try is
_
10
8
_
_
90
8
_.
Problem 48. A standard 52-card deck is distributed between 4 players (as in bridge).
Find the probability that:
(a) Player 1 gets all 13 spades.
(b) Some player gets all 13 spades.
Solution.(a) P(A) =
1
(
52
13
)
= 1.57 10
12
.
(b) P(B) = 4P(A) =
4
(
52
13
)
.
23
Homework 3.8
Homework 3.9

S-ar putea să vă placă și